Catalog of Question types Flashcards
(18 cards)
Identify the conclusion
Which one of the following most accurately expresses the conclusion drawn in the argument?
The conclusion drawn in Annie’s argument is that
Identify an entailment (also known as implication)
Which one of the following can be properly inferred from the statements above?
If the essayist’s statements are true, then which one of the following must also be true?
Which one of the following statements follows logically from the information above?
If the statements above are true, which one of the following CANNOT be true?
Infer what is most strongly supported
Which one of the following most logically completes the argument?
Note: These question stems always feature a blank space like this _______ at the end of the passage.
Laura’s statements above, if true, most strongly support which one of the following hypotheses?
Which one of the following is most strongly supported by the information above?
Which one of the following can most reasonably be concluded on the basis of the information above?
Of the following claims, which one can most justifiably be rejected on the basis of the statements above?
Identify or infer an issue in dispute
X and Y disagree over whether
X and Y disagree with each other about which one of the following?
The dialogue provides the most support for the claim that X and Y disagree over whether
On the basis of their statements, X and Y are committed to disagreeing over the truth of which one of the following statements?
Identify the technique
The educator’s argument proceeds by
Paul responds to Sara’s argument using which one of the following argumentative techniques?
Which one of the following is a technique of reasoning used in the argument?
X responds to Y’s argument by doing which one of the following?
Identify the role
The claim that … plays which one of the following roles in the argument?
The claim that … is used in the argument to
Which one of the following most accurately describes the role played in the argument by the claim that …?
Identify the principle
Which one of the following principles, if valid, most helps to justify the reasoning in the argument above?
The journalist’s reasoning most closely conforms to which one of the following principles?
The situation described above conforms most closely to which one of the following generalizations?
The principle stated above, if valid, most helps to justify the reasoning in which one of the following arguments?
Match the structure
Which one of the following arguments is most similar in its reasoning to the argument above?
The pattern of reasoning in the argument above is most similar to that in which one of the following arguments?
Which one of the following arguments is most closely parallel in its reasoning to the argument above?
Match principles
Which one of the following conforms most closely to the principle illustrated above?
The principle underlying the argument above is most similar to the principle underlying which one of the following arguments?
Identify a flaw
The reasoning in the magazine article’s argument is flawed in that the argument
Which one of the following most accurately describes a flaw in the argument’s reasoning?
The argument commits which one of the following errors of reasoning?
The reasoning in the argument is most vulnerable to criticism on the grounds that the argument
Match flaws
The flawed pattern of reasoning in the argument above is most similar to that in which one of the following?
Which one of the following arguments is most similar in its flawed reasoning to the argument above?
Which one of the following arguments contains flawed reasoning that is most parallel to that in the argument above?
The flawed nature of the argument above can most effectively be demonstrated by noting that, by parallel reasoning, we could conclude that
Necessary Assumptions
Which one of the following is an assumption required by the argument?
The argument requires assuming which one of the following?
The argument requires the assumption that
The argument relies on assuming which one of the following?
Which one of the following is an assumption on which the argument depends?
Sufficient Assumptions
The conclusion drawn above follows logically if which one of the following is assumed?
The conclusion of the argument is strongly supported if which one of the following is assumed?
The conclusion of X’s argument can be properly drawn if which one of the following is assumed?
Which one of the following is an assumption that, if true, would do most to justify X’s actions?
Strengthen
Which one of the following, if true, most strengthens the argument?
Which one of the following, if true, most strengthens the support for the scientist’s hypothesis?
Which one of the following, if true, adds the most support for the conclusion of the argument?
Each of the following, if true, supports the claim above EXCEPT
Weaken
Which one of the following, if true, most seriously weakens the argument?
Which one of the following, if true, most calls into question the efficacy of the traditional treatment described above?
Which one of the following, if true, is the strongest logical counter that the linguist can make to the philosopher?
Which one of the following, if true, most undermines the claim made above?
Identify what is most/least helpful to know
Which one of the following would be most useful to know in order to evaluate the argument?
The answer to which one of the following questions would most help in evaluating the argument above?
The answer to which one of the following questions would LEAST help in evaluating the argument?
Explain
Which one of the following, if true, most helps to explain the apparently paradoxical result?
Which one of the following, if true, most helps to explain the failure of the strategy?
Which one of the following, if true, most helps to explain the preference described above?
Each of the following, if true, contributes to an explanation of the difference in caloric intake EXCEPT:
Resolve a conflict
Which one of the following, if true, most helps to resolve the apparent conflict described above?
Which one of the following, if true, most helps to resolve the apparent discrepancy in the information above?
Which one of the following, if true, does most to justify the doctors’ apparently paradoxical belief?